You are on page 1of 21

2

2.1

Continuity, Dierentiability and Taylors theorem


Limits of real valued functions

Let f (x) be dened on (a, b) except possibly at x0 .


Denition 2.1.1. We say that lim f (x) = L if, for every real number  > 0, there exists
a real number > 0 such that

xx0

0 < |x x0 | < = |f (x) L| < .

(2.1)

Equivalently,
Remark 2.1. The above denition is equivalent to: for any sequence {xn } with xn x0 ,
we have f (xn ) L as n .
Proof. Suppose limxx0 f (x) exists. Take  > 0 and let {xn } be a sequence converging
to x0 . Then there exists N such that |xn x0 | < for n N . Then by the denition
|f (xn ) L| < . i.e., f (xn ) L.
For the other side, assume that xn c = f (xn ) L. Suppose the limit does not
exist. i.e., 0 > 0 such that for any > 0, and all |x x0 | < , we have |f (x) L| 0 .
Then take = n1 and pick xn in |xn x0 | < n1 , then xn x0 but |f (xn ) L| 0 . Not
possible.
Theorem 2.1.2. If limit exists, then it is unique.
Proof. Proof is easy.
3x
1
1) = . Let  > 0. We have to nd > 0 such that (2.1) holds
x1 2
2
with L = 1/2. Working backwards,
Examples: (i) lim (

2
3
|x 1| <  whenever |x 1| < := .
2
3

x2 x = 2
(ii) Prove that lim f (x) = 4, where f (x) =
x2
1 x=2
1
Problem: Show that lim sin( ) does not exist.
x0
x
1
1
Consider the sequences {xn } = { }, {yn } = {
}. Then it is easy to see that
n
2n + 2
 
 
1
1
xn , yn 0 and sin
0, sin
1. In fact, for every c [1, 1], we can nd
xn
yn

a sequence zn such that zn 0 and sin(

1
) c as n .
zn

By now we are familiar with limits and one can expect the following:
Theorem 2.1.3. Suppose lim f (x) = L and lim g(x) = M , then
xc

xc

1. lim(f (x) g(x)) = L M .


xc

2. f (x) g(x) for all x in an open interval containing c. Then L M .


f
L
3. lim(f g)(x) = LM and when M = 0, lim (x) =
.
xc
xc g
M
4. (Sandwich): Suppose that h(x) satises f (x) h(x) g(x) in an interval containing c, and L = M . Then lim h(x) = L.
xc

Proof. We give the proof of (iii). Proof of other assertions are easy to prove. Let  > 0.
From the denition of limit, we have 1 , 2 , 3 > 0 such that
|x c| < 1 = |f (x) L| <

1
= |f (x)| < N for some N > 0,
2


, and
2M

.
|x c| < 3 = |g(x) M | <
2N
Hence for |x c| < = min{1 , 2 , 3 }, we have
|x c| < 2 = |f (x) L| <

|f (x)g(x) LM | |f (x)g(x) f (x)M | + |f (x)M LM |


|f (x)||g(x) M | + M |f (x) L|
< .
To prove the second part, note that there exists an interval (c , c + ) around c such
that g(x) = 0 in (c , c + ).
Examples: (i) lim xm = 0 (m > 0). (ii) lim x sin x = 0.
x0

x0

Remark: Suppose f (x) is bounded in an interval containing c and lim g(x) = 0. Then
xc

lim f (x)g(x) = 0.

xc

Examples: (i) lim |x| sin


x0

1
= 0.
x

(ii) limx0 |x| ln |x| = 0.

One sided limits: Let f (x) is dened on (c, b). The right hand limit of f (x) at c is L,
if given  > 0, there exists > 0, such that
x c < = |f (x) L| < .
Notation: lim+ f (x) = L. Similarly, one can dene the left hand limit of f (x) at b and is
xc

denoted by lim f (x) = L.


xb

Both theorems above holds for right and left limits. Proof is easy.
Problem: Show that lim0 sin = 1.
Solution: Consider the unit circle centered at O(0, 0) and passing through A(1, 0) and
B(0, 1). Let Q be the projection of P on xaxis and the point T is such that A is the
projection of T on xaxis. Let OT be the ray with AOT = , 0 < < /2. Let P be
the point of intersection of OT and circle. Then OP Q and OT A are similar triangles
and hence, Area of OAP < Area of sector OAP < area of OAT . i.e.,
1
1
1
sin < < tan
2
2
2
dividing by sin , we get 1 >

sin
sin
> cos . Now lim+ cos = 1 implies that lim+
= 1.
0
0

sin
Now use the fact that
is even function.

At this stage, it is not dicult to prove the following:

Theorem 2.1.4. limxa f (x) = L exists limxa+ f (x) = limxa f (x) = L.


Limits at innity and innite limits
Denition 2.1.5. f (x) has limit L as x approaches +, if for any given  > 0, there
exists M > 0 such that
x > M = |f (x) L| < .
Similarly, one can dene limit as x approaches .
1
Problem: (i) lim = 0, (ii) limx x1 = 0. (iii) lim sin x does not exist.
x x
x
Solution: (i) and (ii) are easy. For (iii), Choose xn = n and yn = 2 + 2n. Then
xn , yn and sin xn = 0, sin yn = 1. Hence the limit does not exist.
Above two theorems on limits hold in this also.

Denition 2.1.6. (Horizontal Asymptote:) A line y = b is a horizontal asymptote of


y = f (x) if either lim f (x) = b or lim f (x) = b.
x

Examples: (i) y = 1 is a horizontal asymptote for 1 +

1
x+1

Denition 2.1.7. (Innite Limit): A function f (x) approaches (f (x) ) as x


x0 if, for every real B > 0, there exists > 0 such that
0 < |x x0 | < = f (x) > B.
Similarly, one can dene for . Also one can dene one sided limit of f (x) approaching
or .
1
Examples (i) lim 2 = , (ii) limx0 x12 sin( x1 ) does not exist.
x0 x
For (i) given B > 0, we can choose
that sin x1n = 1, say

1
xn

+ 2n and

1 .
B

1
yn

lim f (yn ) = 0, though xn , yn 0 as n .

For (ii), choose a sequence {xn } such


1
= n. Then lim f (xn ) = 2 and
n
xn

Denition 2.1.8. (Vertical Asymptote:) A line x = a is a vertical asymptote of y = f (x)


if either lim+ f (x) = or lim f (x) = .
xa

xa

x+3
.
x+2

Example: f (x) =
x = 2 is a vertical asymptote and y = 1 is a horizontal asymptote.

2.2

Continuous functions

Denition 2.2.1. A real valued function f (x) is said to be continuous at x = c if


(i) c domain(f )
(ii) lim f (x) exists
xc

(iii) The limit in (ii) is equal to f (c).


In other words, for every sequence xn c, we must have f (xn ) f (c) as n .
i.e., for a given  > 0, there exists > 0 such that
0 < |x c| < = |f (x) f (c)| < .

Examples: (i) f (x) =

x2 sin( x1 ) x = 0
0

x=0

is continuous at 0.

Let  > 0. Then |f (x) f (0)| |x2 |. So it is enough to choose =



(ii) g(x) =
Choose

1
xn

1
x

0
=

sin x1

.

x = 0

is not continuous at 0.
x=0
+ 2n. Then lim xn = 0 and f (xn ) =

1
xn

The following theorem is an easy consequence of the denition.


Theorem 2.2.2. Suppose f and g are continuous at c. Then
(i) f g is also continuous at c
(ii) f g is continuous at c
f
(iii) is continuous at c if g(c) = 0.
g
Theorem 2.2.3. Composition of continuous functions is also continuous i.e., if f is
continuous at c and g is continuous at f (c) then g(f (x)) is continuous at c.
Corollary: If f (x) is continuous at c, then |f | is also continuous at c.
Theorem 2.2.4. If f, g are continuous at c, then max(f, g) is continuous at c.
Proof. Proof follows from the relation
1
1
max(f, g) = (f + g) + |f g|
2
2
and the above theorems.
Types of discontinuities
Removable discontinuity: f (x) is dened every where in an interval containing a except at x = a and limit exists at x = a OR f (x) is dened also at x = a and limit is NOT
equal to function value at x = a. Then we say that f (x) has removable discontinuity at
x = a. These functions can be extended as continuous functions by dening the value of
f to be the limit value at x = a.

Example: f (x) =

sin x
x

x = 0

x=0

. Here limit as x 0 is 1. But f (0) is dened to be 0.

Jump discontinuity: The left and right limits of f (x) exists but not equal. This type
of discontinuities are also called discontinuities of rst kind.


Example: f (x) =

x0

1 x 0

. Easy to see that left and right limits at 0 are dierent.

Innite discontinuity: Left or right limit of f (x) is or .


Example: f (x) =

1
x

has innite discontinuity at x = 0.

Discontinuity of second kind: If either lim f (x) or lim+ f (x) does not exist, then c
xc

xc

is called discontinuity of second kind.


Example: Consider the function

f (x) =

0 xQ
1 x  Q

Then f does not have left or right limit any point c. Indeed, f (c + n1 ) and f (c + n )
converges to dierent values.
Properties of continuous functions
Denition 2.2.5. (Closed set): A subset A of IR is called closed set if A contains all its
limit points. (i.e., if {xn } A and xn c, then c A).
Theorem 2.2.6. Continuous functions on closed, bounded interval is bounded.
Proof. Let f (x) be continuous on [a, b] and let {xn } [a, b] be a sequence such that
|f (xn )| > n. Then {xn } is a bounded sequence and hence there exists a subsequence
{xnk } which converges to c. Then f (xnk ) f (c), a contradiction to |f (xnk | > nk .
Theorem 2.2.7. Let f (x) be a continuous function on closed, bounded interval [a, b].
Then maximum and minimum of functions are achieved in [a, b].
Proof. Let {xn } be a sequence such that f (xn ) max f . Then {xn } is bounded and hence
by Bolzano-Weierstrass theorem, there exists a subsequence xnk such that xnk x0 for
some x0 . a xn b implies x0 [a, b]. Since f is continuous, f (xnk ) f (x0 ). Hence
f (x0 ) = max f . The attainment of minimum can be proved by noting that f is also
continuous and min f = max(f ).
Remark: Closed and boundedness of the interval is important in the above theorem.
Consider the examples (i) f (x) = x1 on (0, 1) (ii) f (x) = x on IR.

Theorem 2.2.8. Let f (x) be a continuous function on [a, b] and let f (c) > 0 for some
c (a, b), Then there exists > 0 such that f (x) > 0 in (c , c + ).
Proof. Let  = 12 f (c) > 0. Since f (x) is continuous at c, there exists > 0 such that
1
|x c| < = |f (x) f (c)| < f (c)
2
i.e., 12 f (c) < f (x) f (c) < 12 f (c). Hence f (x) > 12 f (c) for all x (c , c + ).
Corollary: Suppose a continuous functions f (x) satises
tinuous functions (x) on [a, b]. Then f (x) 0 on [a, b].

b
a

f (x)(x)dx = 0 for all con-

Proof. Suppose f (c) > 0. Then by above theorem f (x) > 0 in (c , c + ). Choose
b
(x) so that (x) > 0 in (c /2, c + /2) and is 0 otherwise. Then a f (x)(x) > 0. A
contradiction.
Alternatively, one can choose (x) = f (x).
Theorem 2.2.9. Let f (x) be a continuous function on IR and let f (a)f (b) < 0 for some
a, b. Then there exits c (a, b) such that f (c) = 0.
Proof. Assume that f (a) < 0 < f (b). Let S = {x [a, b] : f (x) < 0}. Then [a, a + ) S
for some > 0 and S is bounded. Let c = sup S. We claim that f (c) = 0. Take
xn = c + n1 , then xn  S, xn c. Therefore, f (c) = lim f (xn ) 0. On the otherhand,
taking yn = c n1 , we see that yn S for n large and yn c, f (c) = lim f (yn ) 0.
Hence f (c) = 0.
Corollary: Intermediate value theorem: Let f (x) be a continuous function on [a, b]
and let f (a) < y < f (b). Then there exists c (a, b) such that f (c) = y
Remark: A continuous function assumes all values between its maximum and minimum.
Problem: (xed point): Let f (x) be a continuous function from [0, 1] into [0, 1]. Then
show that there is a point c [0, 1] such that f (c) = c.
Dene the function g(x) = f (x) x. Then g(0) 0 and g(1) 0. Now Apply Intermediate value theorem.
Application: Root nding: To nd the solutions of f (x) = 0, one can think of dening
a new function g such that g(x) has a xed point, which in turn satises f (x) = 0.
 10 1/2
Example: (1) f (x) = x3 + 4x2 10 in the interval [1, 2]. Dene g(x) = 4+x
. We can
check that g maps [1, 2] into [1, 2]. So g has xed point in [1, 2] which is also solution
7

of f (x) = 0. Such xed points can be obtained as limit of the sequence {xn }, where
xn+1 = g(xn ), x0 (1, 2). Note that

g (x) =

10
1
< .
3/2
(4 + x)
2

By Mean Value Theorem, z (see next section)


1
|xn+1 xn | = |g  (z)||xn xn1 | |xn xn1 |
2
Iterating this, we get
1
|xn+1 xn | < n |x1 x0 |.
2
Therefore, {xn } is a Cauchy sequence. (see problem after Theorem 1.4.4).
Uniformly continuous functions
Denition: A function f (x) is said to be uniformly continuous on a set S, if for given
 > 0, there exists > 0 such that
x, y S, |x y| < = |f (x) f (y)| < .
Here depends only on , not on x or y.
Proposition: If f (x) is uniformly continuous function for ANY two sequences
{xn }, {yn } such that |xn yn | 0, we have |f (xn ) f (yn )| 0 as n .
Proof. Suppose not. Then there exists {xn }, {yn } such that |xn yn | 0 and |f (xn )
f (yn )| > for some > 0. Then it is clear that for  = , there is no for which
|x y| < = |f (x) f (y)| < . Because the above sequence satises |x y| < , but
its image does not.
For converse, assume that for any two sequences {xn }, {yn } such that |xn yn | 0 we
have |f (xn ) f (yn )| 0. Suppose f is not uniformly continuous. Then by the denition
there exists 0 such that for any > 0, |x y| < = |f (x) f (y)| > 0 . Now
take = n1 and choose xn , yn such that |xn yn | < 1/n. Then |f (xn ) f (yn )| > 0 . A
contradiction.
Examples: (i) f (x) = x2 is uniformly continuous on bounded interval [a, b].
Note that |x2 y 2 | |x + y||x y| 2b|x y|. So one can choose < 2b .
8

(ii) f (x) = x1 is not uniformly continuous on (0, 1).


1
Take xn = n+1
, yn = n1 , then for n large |xn yn | 0 but |f (xn ) f (yn )| = 1.
(iii) f (x) = x2 is not uniformly continuous on IR.
Take xn = n + n1 and yn = n. Then |xn yn | = n1 0, but |f (xn ) f (yn )| = 2 +

1
n2

> 2.

Remarks:
1. It is easy to see from the denition that if f, g are uniformly continuous, then f g
is also uniformly continuous.
2. If f, g are uniformly continuous, then f g need not be uniformly continuous. This
can be seen by noting that f (x) = x is uniformly continuous on IR but x2 is not
uniformly continuous on IR.
Theorem 2.2.10. A continuous function f (x) on a closed, bounded interval [a, b] is
uniformly continuous.
Proof. Suppose not. Then there exists  > 0 and sequences {xn } and {yn } in [a, b] such
that |xn yn | < n1 and |f (xn ) f (yn )| > . But then by Bolzano-Weierstrass theorem,
there exists a subsequence {xnk } of {xn } that converges to x0 . Also ynk x0 . Now since
f is continuous, we have f (x0 ) = lim f (xnk ) = lim f (ynk ). Hence |f (xnk ) f (ynk )| 0,
a contradiction.
Corollary: Suppose f (x) has only removable discontinuities in [a, b]. Then f, the extension of f , is uniformly continuous.
Example: f (x) = sinx x for x = 0 and 0 for x = 0 on [0, 1].
Theorem 2.2.11. Let f be a uniformly continuous function and let {xn } be a cauchy
sequence. Then {f (xn )} is also a Cauchy sequence.
Proof. Let  > 0. As f is uniformly continuous, there exists > 0 such that
|x y| < = |f (x) f (y)| < .
Since {xn } is a Cauchy sequence, there exists N such that
m, n > N = |xn xm | < .
Therefore |f (xn ) f (xm )| < .

Example: f (x) =

1
x2

The sequence xn =
continuous.

2.3

1
n

is not uniformly continuous on (0, 1).


is cauchy but f (xn ) = n2 is not. Hence f cannot be uniformly

Dierentiability

Denition 2.3.1. A real valued function f (x) is said to be dierentiable at x0 if


f (x0 + h) f (x0 )
h0
h
lim

exists.

This limit is called the derivative of f at x0 , denoted by f  (x0 ).


Example: f (x) = x2
2xh + h2
= 2x.
h0
h
Theorem 2.3.2. If f (x) is dierentiable at a, then it is continuous at a.
f  (x) = lim

Proof. For x = a, we may write,


f (x) = (x a)

f (x) f (a)
+ f (a).
(x a)

(a)
= f  (a), we
Now taking the limit x a and noting that lim(x a) = 0 and lim f (x)f
(xa)
get the result.
f
Theorem 2.3.3. Let f, g be dierentiable at c (a, b). Then f g, f g and (g(c) = 0)
g
is also dierentiable at c

Proof. We give the proof for product formula: First note that
(f g)(x) (f g)(c)
g(x) g(c)
f (x) f (c)
= f (x)
+ g(c)
.
xc
xc
xc
Now taking the limit x c, we get the product formula
(f g) (c) = f (c)g  (c) + f  (c)g(c).
Since g(c) = 0 and g is continuous, we get g(x) = 0 in a small interval around c.
Therefore
f
f
g(c)f (x) g(c)f (c) + g(c)f (c) g(x)f (c)
(x) (c) =
g
g
g(x)g(c)
10

Hence
(f /g)(x) (f /g)(c)
=
xc

f (x) f (c)
g(x) g(c)
g(c)
f (c)
xc
xc

1
g(x)g(c)

Now taking the limit x c, we get


g(c)f  (c) f (c)g  (c)
f
.
( ) (c) =
g
g 2 (c)
Theorem 2.3.4. (Chain Rule): Suppose f (x) is dierentiable at c and g is dierentiable
at f (c), then h(x) := g(f (x)) is dierentiable at c and
h (c) = g  (f (c))f  (c)
Proof. Dene the function h as
 g(y)g(f (c))
h(y) =

yf (c)

y = f (c)

g (f (c))

y = f (c)

Then the function h is continuous at y = f (c) and g(y) g(f (c)) = h(y)(y f (c)), so
g(f (x)) g(f (c))
f (x) f (c)
= h(f (x))
.
xc
xc
Now taking limit x c, we get the required formula.
Local extremum: A point x = c is called local maximum of f (x), if there exists > 0
such that
0 < |x c| < = f (c) f (x).
Similarly, one can dene local minimum: x = b is a local minimum of f (x) if there exists
> 0 such that
0 < |x b| < = f (b) f (x).
Theorem 2.3.5. Let f (x) be a dierentiable function on (a, b) and let c (a, b) is a local
maximum of f . Then f  (c) = 0.
Proof. Let be as in the above denition. Then
x (c, c + ) =

f (x) f (c)
0
xc

x (c , c) =

f (x) f (c)
0.
xc

11

Now taking the limit x c, we get f  (c) = 0.


Theorem 2.3.6. Rolles Theorem: Let f (x) be a continuous function on [a, b] and dierentiable on (a, b) such that f (a) = f (b). Then there exists c (a, b) such that f  (c) = 0.
Proof. If f (x) is constant, then it is trivial. Suppose f (x0 ) > f (a) for some x0 (a, b),
then f attains maximum at some c (a, b). Other possibilities can be worked out similarly.
Theorem 2.3.7. Mean-Value Theorem (MVT): Let f be a continuous function on [a, b]
and dierentiable on (a, b). Then there exists c (a, b) such that
f (b) f (a) = f  (c)(b a).
Proof. Let l(x) be a straight line joining (a, f (a)) and (b, f (b)). Consider the function
g(x) = f (x) l(x). Then g(a) = g(b) = 0. Hence by Rolles theorem
0 = g  (c) = f  (c)

f (b) f (a)
ba

Corollary: If f is a dierentiable function on (a, b) and f  = 0, then f is constant.


Proof. By mean value theorem f (x) f (y) = 0 for all x, y (a, b).
Example: Show that | cos x cos y| |x y|.
Use Mean-Value theorem and the fact that | sin x| 1.
Problem: If f (x) is dierentiable and sup |f  (x)| < C for some C. Then, f is uniformly
continuous.
Apply mean value theorem to get |f (x) f (y)| C|x y| for all x, y.
Denition: A function f (x) is strictly increasing on an interval I, if for x, y I with
x < y we have f (x) < f (y). We say f is strictly decreasing if x < y in I implies
f (x) > f (y).
Theorem 2.3.8. A dierentiable function f is (i) strictly increasing in (a, b) if f  (x) > 0
for all x (a, b). (ii) strictly decreasing in (a, b) if f  (x) < 0.
Proof. Choose x, y in (a, b) such x < y. Then by MVT, for some c (x, y)
f (x) f (y)
= f  (c) > 0.
xy
Hence f (x) < f (y).

12

2.4

Taylors theorem and Taylor Series

Let f be a k times dierentiable function on an interval I of IR. We want to approximate


this function by a polynomial Pn (x) such that Pn (a) = f (a) at a point a. Moreover, if
the derivatives of f and Pn also equal at a then we see that this approximation becomes
more accurate in a neighbourhood of a. So the best coecients of the polynomial can
(k)
be calculated using the relation f (k) (a) = Pn (a), k = 0, 1, 2, ..., n. The best is in the
sense that if f (x) itself is a polynomial of degree less than or equal to n, then both f
n

f (k) (a)
and Pn are equal. This implies that the polynomial is
(x a)k . Then we write
k!
k=0
f (x) = Pn (x) + Rn (x) in a neighbourhood of a. From this, we also expect the Rn (x) 0
as x a. In fact, we have the following theorem known as Taylors theorem:
Plot of sin(x) and its Taylor polynomials
4
x

3
2
1
sin(x)

xx /3!+x /5!
sin(x)

0
1

xx3/6

2
3
4

pi

pi/2

0
x

pi/2

pi

Figure 1: Approximation of sin(x) by Taylors polynomials

Theorem 2.4.1. Let f (x) and its derivatives of order m are continuous and f (m+1) (x)
exists in a neighbourhood of x = a. Then there exists c (a, x)( or c (x, a)) such that
f (x) = f (a) + f  (a)(x a) + ..... + f (m) (a)
where Rm (x) =

f (m+1) (c)
(x a)m+1 .
(m + 1)!

13

(x a)m
+ Rm (x)
m!

Proof. Dene the functions F and g as


F (y) = f (x) f (y) f  (y)(x y) ....

g(y) = F (y)

xy
xa

f (m) (y)
(x y)m ,
m!

m+1
F (a).

Then it is easy to check that g(a) = 0. Also g(x) = F (x) = f (x) f (x) = 0. Therefore,
by Rolles theorem, there exists some c (a, x) such that
g  (c) = 0 = F  (c) +

(m + 1)(x c)m
F (a).
(x a)m+1

On the other hand, from the denition of F ,


F  (c) =
Hence F (a) =

f (m+1) (c)
(x c)m .
m!

(x a)m+1 (m+1)
f
(c) and the result follows.
(m + 1)!

Examples:

xn
x 2 x3
+
+ .... + ec , c (0, x) or (x, 0) depending on the sign of x.
2!
3!
n!
xn
n
x 3 x5
+
+ .... +
sin(c +
), c (0, x) or (x, 0).
(ii) sin x = x
3!
5!
n!
2
2
4
n
x
x
n
x
+
+ .... +
cos(c +
), c (0, x) or (x, 0).
(iii) cos x = 1
2!
4!
n!
2

(i) ex = 1 + x +

Problem: Find the order n of Taylor Polynomial Pn , about x = 0 to approximate ex in


(1, 1) so that the error is not more than 0.005
Solution: We know that pn (x) = 1 + x + .... +
|Rn (x)|
So n is such that

e
(n+1)!

xn
.
n!

The maximum error in [-1,1] is

1
e
max |x|n+1 ex
.
(n + 1)! [1,1]
(n + 1)!

0.005 or n 5.

Problem: Find the interval of validity when we approximate cos x with 2nd order polynomial with error tolerance 104 .
2

Solution: Taylor polynomial of degree 2 for cos x is 1 x2 . So the remainder is (sin c) x3! .
14

Since | sin c| 1, the error will be atmost 104 if | x3! | 104 . Solving this gives |x| < 0.084
Taylors Series
Suppose f is innitely dierentiable at a and if the remainder term in the Taylors formula,
Rn (x) 0 as n . Then we write
f (x) =

f (n) (a)
n=0

n!

(x a)n .

This series is called Taylor series of f (x) about the point a.


Suppose there exists C = C(x) > 0, independent of n, such that |f (n) (x)| C(x). Then
|x a|n+1
= 0. For any xed x and a, we can always nd N such
|Rn (x)| 0 if lim
n (n + 1)!
that |x a| < N . Let q := |xa|
< 1. Then
N







(x a)n+1 |x a| |x a| |x a| |x a| |x a|
=

...

...


(n + 1)! 1 2 N 1 N n + 1


|x a|N 1 nN +2

q
<
(N 1)!
0 as n thanks to q < 1.
In case of a = 0, the formula obtained in Taylors theorem is known as Maclaurins formula and the corresponding series that one obtains is known as Maclaurins series.
Example: (i) f (x) = ex .

xn+1 (n+1)
xn+1 c
xn+1 x
f
e =
e , for some (0, 1).
(c) =
(n + 1)!
(n + 1)!  (n + 1)!

xn+1
ex = 0.
Therefore for any given x xed, lim |Rn (x)| = lim
n
n
(n + 1)!

In this case Rn (x) =

Example: (ii) f (x) = sin x.


In this case it is easy to see that |Rn (x)|
| sin x| 1 and follow as in example (i).

|x|2n+1
n
| sin(c +
)|. Now use the fact that
(2n + 1)!
2

Maxima and Minima: Derivative test


Denition 2.4.2. A point x = a is called critical point of the function f (x) if f  (a) = 0.

15

Second derivative test: A point x = a is a local maxima if f  (a) = 0, f  (a) < 0.


Suppose f (x) is continuously dierentiable in an interval around x = a and let x = a be
a critical point of f . Then f  (a) = 0. By Taylors theorem around x = a, there exists,
c (a, x) (or c (x, a)),
f  (c)
f (x) f (a) =
(x a)2 .
2
If f  (a) < 0. Then by Theorem2.2.8, f  (c) < 0 in |x a| < . Hence f (x) < f (a) in
|x a| < , which implies that x = a is a local maximum.
Similarly, one can show the following for local minima: x = a is a local minima if
f  (a) = 0, f  (a) > 0.
Also the above observations show that if f  (a) = 0, f  (a) = 0 and f (3) (a) = 0, then the
sign of f (x) f (a) depends on (x a)3 . i.e., it has no constant sign in any interval
containing a. Such point is called point of inection or saddle point.
We can also derive that if f  (a) = f  (a) = f (3) (a) = 0, then we again have x = a is a
local minima if f (4) (a) > 0 and is a local maxima if f (4) (a) < 0.
Summarizing the above, we have:
Theorem 2.4.3. Let f be a real valued function that is dierentiable 2n times and f (2n)
is continuous at x = a . Then
1. If f (k) (a) = 0 for k = 1, 2, ....2n 1 and f (2n) (a) > 0 then a is a point of local
minimum of f (x)
2. If f (k) (a) = 0 for k = 1, 2, ....2n1 and f (2n) (a) < 0 then a is a point local maximum
of f (x).
3. If f (k) = 0 for k = 1, 2, ....2n 2 and f (2n1) (a) = 0, then a is point of inection.
i.e., f has neither local maxima nor local minima at x = a.
LHospitals Rule:
Suppose f (x) and g(x) are dierentiable n times, f (n) , g (n) are continuous at a and
f (k) (a) = g (k) (a) = 0 for k = 0, 1, 2, ...., n 1. Also if g (n) (a) = 0. Then by Taylors

16

theorem,
f (x)
f (n) (c)
= lim (n)
xa g(x)
xa g
(c)
(n)
f (a)
= (n)
g (a)
lim

In the above, we used the fact that g (n) (x) = 0 near x = a and g (n) (c) g (n) (a) as
x a.
Similarly, we can derive a formula for limits as x approaches innity by taking x = y1 .
lim

2.5

f (x)
f (1/y)
= lim
g(x) y0 g(1/y)
(1/y 2 )f  (1/y)
= lim
y0 (1/y 2 )g  (1/y)
f  (x)
= lim 
x g (x)

Power series & Taylor Series

Given a sequence of real numbers {an }


n=0 , the series


n=0

an (x c)n is called power series

with center c. It is easy to see that a power series converges for x = c. Power series is a
function of x provided it converges for x. If a power series converges, then the domain of
convergence is either a bounded interval or the whole of IR. So it is natural to study the
largest interval where the power series converges.


an xn converges for |x| < |r|.
Remark: If
an xn converges at x = r, then
Proof: We can nd C > 0 such that |an xn | C for all n. Then
x
x
|an xn | |an rn || |n C| |n .
r
r
Conclusion follows from comparison theorem.
Theorem 2.5.1. Consider the power series
R=
1.

n=0

an xn . Suppose = lim sup

(We dene R = 0 if = and R = if = 0). Then

n=0

2.


n=0

an xn converges for |x| < R


an xn diverges for |x| > R.

3. No conclusion if |x| = R.
17


n
|an | and

Proof. Proof of (i) follows from the root test. For a proof, take n (x) = an xn and
= lim sup n |n |. For (ii), one can show that if |x| > R, then there exists a subsequence
{an } such that an  0. Notice that = |x|. For (iii), observe as earlier that the series
with an = n1 and bn = n12 will have R = 1.
Similarly, we can prove:




and
an xn . Suppose = lim sup an+1
Theorem 2.5.2. Consider the power series
an
R=
1.

n=0

(We dene R = 0 if = and R = if = 0). Then

n=0

2.


n=0

an xn converges for |x| < R


an xn diverges for |x| > R.

3. No conclusion if |x| = R.
Denition 2.5.3. The real number R in the above theorems is called the Radius of convergence of power series.
n
n 3n
xn
(ii) xn! (iii)
2 x
Examples: Find the interval of convergence of (i)
n
1. = lim sup | an+1
| = 1, and we know that the series does not converge for x = 1, 1.
an
So the interval of convegence is (1, 1).
2. = lim sup | an+1
| = 0. Hence the series converges everywhere.
an
n n

2 y .
3. To see the subsequent non-zero
terms, we write the series as 2n (x3 )n =

n
1
1/3
1/3
For this series y = lim sup |an | = 2 . Therefore, x = 2
and R = 2 .
The following theorem is very useful in identifying the domain of convergence of some
Taylor series.


an x n
Theorem 2.5.4. (Term by term dierentiation and integration): Suppose f (x) =
n=0

converges for |x| < R. Then


1.


n=0

2.


n=0

nan xn1 converges in |x| < R and is equal to f  (x).


1
a xn+1
n+1 n

converges in |x| < R and is equal to

f (x)dx.

From this theorem one concludes that a power series is innitely dierentiable with in
its radius of convergence. Now it is natural to ask weather this series coincides with the
Taylor series of the resultant function. The answer is yes and it is simple to prove that
18


(n)
if f (x) =
an xn , then an = fn! . The above theorem is useful to nd the domain of
convergence of Taylor series of some functions.
Example: The Taylor series of f (x) = tan1 x and a domain of its convergence.


dx
=
1 x2 + x4 + ...
tan1 x =
1 + x2
x3 x 5
+
+ ...
=x
3
5
Taking x = 1 we get interesting sum
1

1 1 1

+ + .... = tan1 (1) = .


3 5 7
4

Though the function tan1 x is dened on all of IR, we see that the power series converges
on (1, 1). We can apply Abels theorem on alternating series to show that the series
converges at x = 1, 1.
For approximation, we can use the error approximation of alternating series discussed
in the previous section. The total error if we approximate tan1 x by sn (x), then the
n+1
maximum error is |x|n+1 .
///
We note that the power series may converge to a function on small interval, even though
the function is dened on a much bigger interval. For example the function log(1 + x) has
power series that converges on (1, 1), but log(1 + x) is dened on (1, ). The domain
of convergence of power series is symmetric about the center but the domain of denition
of function. For instance, for a function dened on (1, 3) the radius of convergence of
its power series(about 0) cannot be more than 1.
Another interesting application is to integrate the functions for which we have no clue.
For example,
 x
 x
x3 x5
t2 t4
t2
(1) erf (x) =
+
+ ...
e dt =
(1 + + ... = x
1! 2!
3
10
0
0


(2)
0

sin t
=
t

x
0

x3
x5
t2 t 4
+ =x
+
+ ...
3! 5!
3! 3 5! 5

19

Problems
1. Determine the points and nature of discontinuity of the following functions:


1 xQ
x4 + 1
cos x
x tan x
1
3x (b)
(c)
(e) f (x) =
(d) 2
(a)
2
x2
x /2
x +1
1 + sin x
0 x  Q
2. Determine if the following equations admits solutions in the interval mentioned.

2x + 1
(a) x5 3x2 = 1, [0, 1] (b) sin2 x2 cos x = 1, [0, ] (c) sin x
, [0, ]
2
x2
3. Bisection Method for nding roots: Let f (x) be a continuous function on [a, b] such
that f (a)f (b) < 0. Then show that f has a zero in [a, b]. Begin with a1 = a, b1 = b
1
and x1 = a1 +b
. If f (x1 ) = 0, we are done. Otherwise, apply the method on (x1 , b1 )
2
2
or (a1 , x1 ). Dene x2 = a2 +b
....and so on..Show that {xn } converges.
2
4. Determine which of the following functions are uniformly continuous in the interval
mentioned.
2

(a) ex sin(x2 ), (0, 1) (b) | sin x|, [0, ) (c)

x sin x


5. Determine
 1 if the following functionsare dierentiable at 0. Find f (0) if exists
x cos x1 x = 0
e x2 x = 0
(b) e|x| (c)
(a)
0
x=0
0
x=0

6. Determine
if f  is continuousat 0 for the following 
functions:

1
1
1
3
2
x2 ln |x|
x = 0
x cos x x =
 0
 0
x sin x x =
(b)
(c)
(a)
0
x=0
0
x=0
0
x=0
7. When a circular plate of metal is heated in an oven, its radius increases at the rate
of 0.01cm/min. At what rate the plates area increasing when the radius is 50 cm?
8. Show the following inequalities:
(a) for 0 < p < 1 and a, b 0, (a + b)p ap + bp .
(b) for p > 1, and a, b 0, (a + b)p 2p1 (ap + bp ).
9. Let f be dierentiable on IR and sup |f  (x)| < 1. Select s0 IR and dene sn =
IR

f (sn1 ). Prove that {sn } is a convergent sequence.

20

10. Let f be a thrice dierentiable


function
on (a, b) such that f (p) = 0, f  (p) = 0 for



f (x)f (x)
< 1. The following sequences come from the
some p (a, b) and sup
f  (x)2
(a,b)
recursion formula for Newtons method:
xn+1 = xn

f (xn )
f  (xn )

Show that the sequence {xn } converges. In each case, begin by identifying the
function f that generates the sequence and determine if the sequence converges:
x0 = 1,
(a) xn+1 = xn

1
xn
+
2
xn

(b) xn+1 = xn 1 (c) xn+1 = xn

tan xn 1
sec2 xn

11. Let f be dierentiable on IR and |f (x) f (y)| (x y)2 . Then show that f is
constant.
12. Prove that if f, g are dierentiable on IR, f  (x) g  (x) on IR and f (0) = g(0), then
f (x) g(x) for x 0.
13. Find the rst 3 terms of the Taylors expansion about the point x0 given
2

(a) sin x, x0 = /2 (b) e1/x , x0 = 0 (c) tan1 x, x0 = 0


14. Show that Rn (x) 0 as n for the following functions
(i) log(1+x) (ii) sinh x (iii) cosh x (iv) ax

(v) (1+x)m , 0 < m Q, x (0, 1)

15. Evaluate the following limits


ex (1 + x)
x0
x2

(a) lim

1 cos t (t2 /2)


t0
t4

(b) lim

(c) lim x2 (e1/x 1)


x

16. Find the domain of validity when sin x is approximated by x (x3 /6) with an error
of magnitude no greater than 5 104
17. Estimate the error in the approximation of sin hx = x + (x3 /3!) when |x| < 0.5.

18. Find the error while approximating 1 + x with 1 + (x/2) in |x| < 0.01.

21

You might also like